Put the following equation of a line into slope-intercept form, simplifying all
fractions.
2y – 2 = -8

Answers

Answer 1

Answer:

[tex]y = x - 4[/tex]

Step-by-step explanation:

Given

[tex]2y - 2x = -8[/tex] -- correct expression

Required

Represent as slope intercept form

[tex]2y - 2x = -8[/tex]

Make 2y the subject

[tex]2y = 2x - 8[/tex]

Divide through by 2

[tex]\frac{2y}{2} = \frac{2x}{2} - \frac{8}{2}[/tex]

[tex]y = x - 4[/tex]


Related Questions

Graph line y=-1/5x-5 with y and x points

Answers

Answer:

Please see picture below.

Step-by-step explanation:

A rectangle is 6 meters long and 4 meters wide. What is the area of the rectangle?

24 cm 2
10 cm 2
48 cm 2
20 cm 2

Answers

The answer is 24 cm squared because to find the area of a rectangle you multiply the width by the height. 6 x 4= 24

30 feet below the surface of the water. You are ascending towards the
surface of the water. The graph models this situation. What is the slope of the line?

Answers

Answer is slope of 5


Step by step


You can graph the slope as seen on the attached picture. You go up 30, you go right +6. Slope is y/x, so 30/6, equals 5



You can also use the slope formula


(y2-y1) over (x2-x1)


Using points on the graph


(6,0) and (0,-30)


(-30-0 ) over (0-6)


-30 over -6


-30/-6


= 5


pls can solve this question . thanks

Answers

the answer is 75

explanation: 180cm divided by 45 cm is 4, so 300cm divided by 4 is 75

The mean date is 1985.67 and standard deviation 9.2 is greater than 2005?

Answers

Answer:

Step-by-step explanation:

Use the mean and the standard deviation obtained from the last discussion and test the claim that the mean age of all books in the library is greater than 2005.

This is the last discussion:

The science portion of my library has roughly 400 books.

They are arranged, on shelves, in order of their Library of Congress

code and, within equal codes, by alphabetical order of author.

Sections Q and QA have a total of 108 books.

The bulk of the library was established in the early 1990s.

I used a deck of cards, removing the jokers and face cards, keeping

only aces (1) and "non-paints" (2 to 10). Starting from the start of

the first shelf, I would turn a card (revealing its number N) and I

would go to the Nth book. This uses ordinal numbers (1 would means the

"first" book, not a gap of 1).

The cards were shuffled sufficiently to assume that the cards have a

random order.

I sampled only the LC subsections Q and QA (therefore, not a true

sample of the entire library, as this classification is not purely

random).

Thus, I picked 21 books (the expected number being 108/5.5 = 19.6 --> 20 books)

The sampled dates of publication were (presented as an ordered set):

1967, 1968, 1969, 1975, 1979, 1983, 1984,

1984, 1985, 1989, 1990, 1990, 1991, 1991,

1991, 1991, 1992, 1992, 1992, 1997, 1999

The median date is 1990

The mean date is 1985.67

Variance = 84.93 ( Sum of (date-mean)^2 )

SQRT of variance = 9.2 (sample standard deviation)

The "sigma-one" confidence interval (containing 68% of the books), if

the sample were NOT skewed, and if the distribution were "normal"

would contain dates from "mean - 9.2" to "mean + 9.2"

Sigma-2 (95%) would have mean - 2(9.2) to mean + 2(9.2)

Sigma-3(99.7%) from "1985.67 - 3(9.2)" to "1985.67 + 3(9.2)"

1958.07 to 2012.6

There, you see one reason why the distribution is skewed (it is

possible to have books older than 1958, but it is impossible to have

book newer than 2012), but it still represents a usable model for the

library. If it applies to the entire library of 400 books, you would

expect one book (0.3% of 400) to fall outside the 3-sigma interval.

Which of the following is not a characteristic of both observational studies and experiments

Answers

Data collected about a population is not a characteristic of both observational studies and experiments.

What is an experiment?

Through experiments, two variables' cause and effect relationships are examined. This is where they differ from observations and interviews, which can only assume the existence of contexts and cannot provide evidence for them. The environment of the test subjects is managed in an experiment depending on the issue.

Three things characterize statistical experiments in general:

There are various outcomes that the experiment could produce.

In an experiment, the cause, the independent variable, is changed while the effect, the dependent variable, is measured and any unrelated factors are controlled.

It is possible to anticipate every outcome.

Chance determines how the experiment turns out.

Learn more about experiments here:

https://brainly.com/question/17274244

#SPJ1

For A (1, –1), B (–1, 3), and C (4, –1), find a possible location of a fourth point, D, so that a parallelogram is formed using A, B, C, D in any order as vertices.

Answers

The possible coordinates for the vertex D of the parallelogram could be (-4, 3)

It is given that A(1, –1), B (–1, 3), and C (4, –1) are the three vertices of the parallelogram.

Let us assume that the vertices are in the order A, C, B, and D where the coordinates of D are (a, b).

In this scenario, if we join AB and CD, they will become the diagonals of the parallelogram ABCD.

According to the properties of a parallelogram, diagonals bisect each other.

Hence, mid-point of AB = mid-point of CD

Now, according to the mid-point theorem, if mid-point of AB is given as (x,y), then,

x = (x₁ + x₂)/2 and y = (y₁ + y₂)/2

Here, for AC,

x₁ = 1, y₁ = -1

x₂ = -1, y₂ = 3

Then, x = ( 1 - 1)/2 and y = (-1 + 3)/2

(x, y) ≡ (0, 1)  ............. (1)

Since (x, y) is also the mid-point of CD, we also have,

x₁ = 4, y₁ = -1

x₂ = a, y₂ = b

Then, x = (4 + a)/2 and y = (-1 + b)/2

(x, y) ≡ ((4 + a)/2, (-1 + b)/2) ................... (2)

From (1) and (2),

(4+a)/2 = 0 and (-1+b)/2 = 1

4+a = 0 and (-1+b) = 2

a = -4 and b = 3

Hence, the fourth vertex D of the parallelogram can be possibly located at (-4, 3)

Learn more about a parallelogram here:

https://brainly.com/question/1563728

#SPJ1

Find the slope-intercept form of the line that
passes through the point (-7, 3) and is
parallel to the line 2x+5y = 3.

Answers

I hope u understand the solution.

For f(x) = 4x+1 and g(x)=x²-5, find (f-g)(x).

Answers

Answer: -x² + 4x + 6

Given:

f(x) = 4x + 1

g(x) = x² - 5

Solve:

= (f - g)(x)

= f(x) - g(x)

= 4x + 1 - (x² - 5)

= 4x + 1 - x² + 5

= -x² + 4x + 6

Answer:

(f-g) (x) = 4x+1 - x² + 5 = -x² + 4x + 6

Step-by-step explanation:

Explanation can be given if needed in the comments (ㅅ´ ˘ `)

Mark the vertex and graph the axis of symmetry of the function.

f(x) = (x – 2)2 – 25

Answers

Answer:

Vertex (2 , -25)

Axis of symmetry: x = 2

Step-by-step explanation:

Vertex of parabola:

 The vertex is the highest point if the parabola open downwards and the lowest point if the parabola opens upward.

 f(x) = (x - 2)² - 25

The given quadratic function  is in vertex form.

 f(x)  = a(x - h)² + k

Here,  (h , k) is the vertex of the parabola.

 h = 2  ; k = -25

[tex]\sf \boxed{\bf Vertex (2 , -25)}[/tex]

Axis of symmetry:

 The axis of symmetry is the vertical line that divides the parabola into two equal halves and it passes through the vertex of the parabola.

Axis of symmetry: x = h

  [tex]\sf \boxed{\bf x = 2}[/tex]

Without exponents
logc (xy^6z^-4) what is the equivalent expression

Answers

By eliminating exponents, the logarithmic expression [tex]\log_{c} x\cdot y^{6}\cdot z^{-4}[/tex] is equivalent to the logarithmic expression [tex]\log_{c} x + 6\cdot \log_{c} y - 4\cdot \log_{c} z[/tex].

How to simplify logarithmic functions

In this problem we are supposed to eliminate all exponents of a logarithmic function by applying any of the following properties:

㏒ x · y = ㏒ x + ㏒ y㏒ x/y = ㏒ x - ㏒ y㏒ yˣ = x · ㏒ y

Now, we proceed to simplify the function:

[tex]\log_{c} x\cdot y^{6}\cdot z^{-4}[/tex]

[tex]\log_{c} x + \log_{c} y^{6} + \log_{c} z^{-4}[/tex]

[tex]\log_{c} x + 6\cdot \log_{c} y - 4\cdot \log_{c} z[/tex]

By eliminating exponents, the logarithmic expression [tex]\log_{c} x\cdot y^{6}\cdot z^{-4}[/tex] is equivalent to the logarithmic expression [tex]\log_{c} x + 6\cdot \log_{c} y - 4\cdot \log_{c} z[/tex].

To learn more on logarithms: https://brainly.com/question/24211708

#SPJ1

The graph below shows the price of different numbers of mats below at a store:

Which equation can be used to determine p, the cost of b mats?

P= 10.50+b
B= 10.50+p
B= 10.50p
P=10.50b

Answers

Answer:

The answer to your question is p = 10.50b

Step-by-step explanation:

Since you want to find p, you're looking for an equation that has

p = <something>

The last selection has this form

p = 10.50b

I hope this helps and have a good day!

Seven more than 4 times a number is 43 solve in algebraic equation?

Answers

Answer:

x = 9

Step-by-step explanation:

4x + 7 = 43

4x = 36

x = 9

find the product of 3.05 and 0.07​

Answers

Answer:

0.2135

Step-by-step explanation:

When something asks to find the product, it indicates multiplication.

We are given two decimals which are: 3.05 and 0.07.

We are asked to multiply and find the product of these decimals.

Let's use the algorithm method.

                   3  .    0   5

x                 0   .    0   7

_____________________

                  2        3

          0 .    2      1    3    5

Therefore, 3.05 × 0.07 = 0.2135.

The figure below shows a triangular piece of cloth: B 35 8 in. What is the length of the portion BC of the cloth? 08 cos 35° sin35° 8 cos 35° 08 sin 35°

Answers

From the calculation, we can see that /BC/ is 8 sin 35. Option D

What is the length of the portion BC of the cloth?

We can see the image of the triangular piece of cloth as shown in the image. This shows that we have to approach the problem by the use of the trigonometric ratios.

Hence;

/AC/ = 8 in

/BC/ = x

<A = 35 degrees

Sin 35 = /BC//8

/BC/ = 8 sin 35

Learn more about trigonometric ratios:https://brainly.com/question/1201366

#SPJ1

A meteorologist found that the rainfall in Vindale during the first half of the month was 1/2 of an inch. At the end of the month, she found that the total rainfall for the month was 2/3 of an inch. How much did it rain in the second half of the month?

Answers

The quantity of rainfall that fell in the second half of the month is; ¹/₆ of an inch.

How to solve fraction word problems?

We are given;

Rainfall during first half of the month = 1/2 inch

Total rainfall for the month = 2/3 inch

Now, to find the rainfall for the second half of the month, we just subtract the one for the first half from the one for the month to get;

Rainfall in second half of the month = ²/₃ - ¹/₂ = ¹/₆ inches

Read more about Fraction Word Problems at; https://brainly.com/question/11562149

#SPJ1

According to the Rational Root Theorem, which statement about f(x) = 12x3 – 5x2 + 6x + 9 is true?
Any rational root of f(x) is a multiple of 12 divided by a multiple of 9.
Any rational root of f(x) is a multiple of 9 divided by a multiple of 12.
Any rational root of f(x) is a factor of 12 divided by a factor of 9.
Any rational root of f(x) is a factor of 9 divided by a factor of 12.

Answers

The complete statement is (d) Any rational root of f(x) is a factor of 9 divided by a factor of 12.

How to determine the rational roots?

The polynomial is given as:

12x^3 - 5x^2 + 6x + 9

The first term in the above equation is 12 i.e. 12x^3, while the last term is 9

To determine the possible rational roots, we divide the factors of the last term i.e. 12 by the factors of the first term i.e. 12

Hence, the complete statement is (d) Any rational root of f(x) is a factor of 9 divided by a factor of 12.

Read more about rational roots at:

brainly.com/question/10937559

#SPJ1

Answer:

D

Step-by-step explanation:

I took the test

HELP!!!!!!!!!!!!!!!!!!!!!!!!

Answers

The recursive formula for f(n) is f(n) = 4.25 + f(n - 1), f(0) = 2.25.

What is an equation?

An equation is an expression that shows the relationship between two or more numbers and variables.

Let f(n) represent the total cost of shoe rentals for n games, hence:

The recursive formula for f(n) is f(n) = 4.25 + f(n - 1), f(0) = 2.25.

Find out more on equation at: https://brainly.com/question/2972832

#SPJ1

The height of a door is 1.5 feet longer than its width, and its front area is 1516.5 square feet. Find the width and height of the door.

Answers

Answer:

For an exact answer the width would be the square root of 1011 and the length would be 1.5x the square root of 1011.

Step-by-step explanation:

A = lw

1516.5 = w(1.5)w

1516.5 = 1.5w^2  Divide both sides by 1.5

1011 = w^2  Take the square root of each side

Square root of 1011 = w.

A bank gets an average of 12 customers per hour. Assume the variable follows a Poisson distribution. Find the probability that there will be 4 or more customers at this bank in one hour.

Answers

Using the Poisson distribution, there is a 0.9978 = 99.78% probability that there will be 4 or more customers at this bank in one hour.

What is the Poisson distribution?

In a Poisson distribution, the probability that X represents the number of successes of a random variable is given by:

[tex]P(X = x) = \frac{e^{-\mu}\mu^{x}}{(x)!}[/tex]

The parameters are:

x is the number of successese = 2.71828 is the Euler number[tex]\mu[/tex] is the mean in the given interval.

A bank gets an average of 12 customers per hour, hence the mean is [tex]\mu = 12[/tex].

The probability that there will be 4 or more customers at this bank in one hour is:

[tex]P(X \geq 4) = 1 - P(X < 4)[/tex]

In which:

P(X < 4) = P(X = 0) + P(X = 1) + P(X = 2) + P(X = 3)

Then:

[tex]P(X = x) = \frac{e^{-\mu}\mu^{x}}{(x)!}[/tex]

[tex]P(X = 0) = \frac{e^{-12}12^{0}}{(0)!} \approx 0[/tex]

[tex]P(X = 1) = \frac{e^{-12}12^{1}}{(1)!} \approx 0[/tex]

[tex]P(X = 2) = \frac{e^{-12}12^{2}}{(2)!} = 0.0004[/tex]

[tex]P(X = 3) = \frac{e^{-12}12^{3}}{(3)!} = 0.0018[/tex]

Then:

P(X < 4) = P(X = 0) + P(X = 1) + P(X = 2) + P(X = 3) = 0 + 0 + 0.0004 + 0.0018 = 0.0022.

[tex]P(X \geq 4) = 1 - P(X < 4) = 1 - 0.0022 = 0.9978[/tex]

0.9978 = 99.78% probability that there will be 4 or more customers at this bank in one hour.

More can be learned about the Poisson distribution at https://brainly.com/question/13971530

#SPJ1

How long do you need to invest your money in an account earning an annual interest rate of 4.252% compounded monthly so that your investment grows from $1,018.40 to $10,413.00 over that period of time?

Answers

The time needed for the investment to become $10,413 is 54.7 years

How long before $1,018.40 grows to $10,413.00?

Number of years = (In FV / PV) / r  

FV = future valuePV = present valuer = interest rate

r = 4.252/12 = 0.354%

(In 10, 413 / 1,018.40) / 0.00354 = 656.73 months = 54.7 years

To learn more about how to determine the number of years an investment would be equal to a certain value, please check: https://brainly.com/question/21841217

#SPJ1

Graph the following inequality. y ≤ 3x +5 Use the graphing tool to graph the inequality Click to enlarge graph​

Answers

Please see the graph below

Seven more than twice a number is six less than three times the same number

Answers

Answer:

13

Step-by-step explanation:

7+2x=3x-6

knowing this then you solve it

Answer: x = 13

Step-by-step explanation:

7 more = + 7

twice a number = 2*x = 2x

6 less = -6

3 times the same number = 3 * x = 3x

Putting this together we get:

2x + 7 = 3x - 6

subtract two from both sides to isolate x

(-2x) + 2x + 7 = 3x - 6 (-2x)

7 = x - 6

add 6 to both sides to get your answer

7 (+6) = x - 6 (+6)

13 = x

or

x = 13

A. Reflection around the x-axis:
1. A (5, 1)→ A'
2. B (1, 3) B¹.
3. C (-2, 2)→ C'.
4. D (-5, 4) D'
B. Reflection around the y-axis:
1. A (5, 1)→ A'
2. B (1, 3) B¹.
3. C (-2, 2)→ C'_
4. D (-5, 4) D'

Answers

Part A

1) (5, -1)

2) (1, -3)

3) (-2, -2)

4) (-5, -4)

Part B

1) (-5, 1)

2) (-1, 3)

3) (2, 2)

4) (5, 4)

Which expression is equivalent to the following complex fraction?

1 minus StartFraction 1 Over x EndFraction divided by 2

Answers

The equivalent expression of [tex]1 - \frac{1}{x} \div 2[/tex] is [tex]\frac{2x - 1}{2x}[/tex]

How to determine the equivalent expression?

The expression is given as:

1 minus StartFraction 1 Over x EndFraction divided by 2

Rewrite properly as:

[tex]1 - \frac{1}{x} \div 2[/tex]

Express the division as product

[tex]1 - \frac{1}{x} \times \frac 12[/tex]

Evaluate the product

[tex]1 - \frac{1}{2x}[/tex]

Take the LCM

[tex]\frac{2x - 1}{2x}[/tex]

Hence, the equivalent expression of [tex]1 - \frac{1}{x} \div 2[/tex] is [tex]\frac{2x - 1}{2x}[/tex]

Read more about equivalent expression at:

https://brainly.com/question/27911936

#SPJ1

Eric's Coffee Shop makes a blend that is a mixture of two types of coffee. Type A coffee costs Eric $5.50 per pound, and type B coffee costs $4.40 per pound. This month's blend used four times as many pounds of type B coffee as type A, for a total cost of $762.30. How many pounds of type A coffee were used?

Answers

115.2 pounds of type A coffee were used.

What is the solution of the equation?

A mathematical statement that has an "equal to" symbol between two expressions with equal values is called an equation.

Equations are mathematical statements with two algebraic expressions flanking the equals (=) sign on either side. It demonstrates the equality of the relationship between the expressions printed on the left and right sides. We have LHS = RHS (left hand side = right hand side) in every mathematical equation. To determine the value of an unknown variable that represents an unknown quantity, equations can be solved.

Let the number of pounds of type A coffee be x and the number of pounds of Type B coffee be y.

According to the question, the equations are,

5.50x + 4.40y = 762.30

x = 4y

So, the solution of the equation is obtained as follows:

5.50(4y) + 4.40y = 762.30

26.40y = 762.30

y = 762.30/26.40

y = 28.8 pounds

x = 4*28.8 = 115.2 pounds

Learn more about equations here:

https://brainly.com/question/2972832

#SPJ1

Find the time required for an investment of 5000 dollars to grow to 6000 dollars at an interest rate of 7.5 percent per year, compounded quarterly.

Answers

Answer:

2.4536 years

Step-by-step explanation:

compound formula:

A=P*(1+r/n)^nt

A = 6000

P = 5000

r = rate

n = 4 times per year compounded

t = time in years

6000 = 5000(1 + .075/4)^4t

divide both sides by 5000

1.2 = (1.01875)^4t

log both sides

log(1.2) = 4t * log(1.01875)

solve for t

t = 2.4537 years

Given that a function, g, has a domain of -20 ≤ x ≤ 5 and a range of -5 ≤ g(x) ≤ 45 and that g(0) = -2 and g(-9) = 6, select the statement that could be true for g.

Answers

A statement which could be true for g is that: A. g(-13) = 20.

What is a domain?

A domain can be defined as the set of all real numbers for which a particular function is completely defined.

How to determine the true statement?

Since the domain of this function is given by -20 ≤ x ≤ 5, it simply means that the value of x must between -20 and 5. Also, with a range of -5 ≤ g(x) ≤ 45, the value of x must between -5 and 45.

By extrapolating the function, we can deduce that:

g(0) = -2g(-9) = 6g(-13) = 20

Read more on domain here: https://brainly.com/question/17003159

#SPJ1

Complete Question:

Given that a function, g, has a domain of -20 ≤ x ≤ 5 and a range of -5 ≤ g(x) ≤ 45 and that g(0) = -2 and g(-9) = 6, select the statement that could be true for g.

A. g(-13) = 20

B. g(-4) = -11

C. g(7) = -1

D.g(0) = 2

Given the following exponential function, identify whether the change
represents growth or decay, and determine the percentage rate of increase or
decrease.
y = 38(1.09)^x

Answers

Step-by-step explanation:

We can determine whether the change represents exponential growth or exponential decay by making a table for a few values of x.

0 -> [tex]38(1.09)^0[/tex] = 38 * 1 = 38

1 -> [tex]38(1.09)^1[/tex]= 38 * 1.09 = 41.42

2 -> 38(1.09)² = 38 * 1.09 * 1.09 = 45.1478

We see that y increases as x increases, which makes this function represent exponential growth.

The percentage rate of increase is how much the value of y increases by each time x increases.

Since we are multiplying by 1.09 each time, we are taking 109% of the previous y value to get the current y value. Hence, the rate of increase is 9%.

Given quadrialateral PQRS is similar to TUVW, find side PS.

Answers

The length of side PS according to the quadrilateral descriptions is; 21.

What is the length of side PS?

Since, it follows from the task content that the two quadrilaterals given are congruent, hence, the corresponding sides are congruent and are in proportion to each other.

On this note, we have;

x/35 = 15/25

x = (35×15)/25

x = 21.

Read more on congruence;

https://brainly.com/question/2102817

#SPJ1

Other Questions
which of the following is considered one of the cons of the plea-bargaining system two cards are selected in a sequence from a standard deck. what is the probability that the second card is a jack given that the first card was a 2. (assume the 2 was not replaced.) A circle is graphed on a coordinate grid with its center at (5, -8). The circle will be translated m units to the left andp units up. Which rule describes the center of the new circle after this translation.:: (x, y) (5 + m, -8 + p)(x, y) (5 m, 8 + p)4-=(x, y) (5 m, -8-p)(x, y) (5+ m, - 8 - p) Flotation cost is thea.cost of holding stock on hand.b.cost of issuing new debt.c.cost of issuing new stock.d.sales price of common stock A double stranded region of RNA that can stall ribosomes is a(n) a. operator sequence b. antiterminator hairpinc. terminator hairpin d. repressor sequence. An auditorium has a volume of 6 x 10^3 m^3. How many molecules of air are needed to fill the auditorium at one atmosphere and 0C? f the current point lies to the ________ of the center of the bullseye, the state of the economy will suggest opposite monetary policy stances. Sugar for domestic use is usually purchased in 2,5kg. Calculate the sugar required in the recipe ,as a percentage,of the quantity usually purchased by a household how many arrangements of inconsistent are there in which ne appear consecutively or no appear consecutively but not both ne and no are consecutive? here is the lamb of god who takes away the sin of the world!"" this highlights jesus role in early tetrapods were evolving limbs so that they could start to crawl on land and take advantace of terrestrial niches TRUE/FALSE Which of the following did Hull believe was true about reaction potential?a. It was an intervening variableb. It would equal zero if either habit strength or drive was zeroc. Both alternatives a. and b.d. None of the above (reaction potential was Tolman's idea under ideal circumstances, it is possible for someone to detect a single drop of perfume diffused in an area the size of a one-bedroom apartment. this is due to Suppose that there are two industries, A and B. There are five firms in industry A with sales at $5 million, $2 million, $1 million, $1 million, and $1 million, respectively. There are four firms in industry B with equal sales of $2.5 million for each firm. The HHI for industry B is:Multiple Choice2,500.1,800.3,200.2,800. which of the following had the most to do with the acceptance today of the notion that the study of individual differences is appropriate subject matter for psychology? Linda is saving money to buy a game. So far she has saved $15, which is three-fifths of the total cost of the game. How much does the game cost? Ridge push and slab pull are both thought to contribute to motion of the tectonic plates. What force drives ng push, slab pull, and convection of the asthenosphere in general A. Convection Cells generated by heat from the Earth's interior B. The Moon's gravitational pull c. Centrifugal force caused by Earth's rotation D. Magnetic forces that originate in Earth's core Consider the following Intel assembly language fragment. Assume that the label my_data refers to a region of writable memory. moveax, my data movebx, Ox0123456 mov [eax), ebx add eax, 2 mov bh, Oxff add [eax), bh add eax, 1 movecx, Oxabcdabcd mov [eax), ecx Give the value of all known memory values starting at my_data. Give your answer as a sequence of hex bytes. Recall that Intel is a little-endian architecture. another student also expected 2.63 g of product, but isolated only 2.45 g. what is the percentage yield? show your work. may be used to mitigate the risk of unauthorized access in computer operations